Showing Rejection Region Equality with Fisher Distribution

Click For Summary
The discussion focuses on demonstrating the equality of rejection regions for testing the equality of variances from two normal populations. It establishes that the rejection region defined by the Fisher distribution can be expressed in two equivalent forms, confirming the relationship between the sample variances. The second part of the problem involves showing that the probability of the larger sample variance over the smaller one exceeding a critical value corresponds to the significance level alpha under the null hypothesis. Participants express confusion regarding the interpretation of tail probabilities in relation to the rejection region. The conversation highlights the need for clarity in understanding the statistical properties of the Fisher distribution and its application in hypothesis testing.
transmini
Messages
81
Reaction score
1

Homework Statement


[/B]
For reference:
Book: Mathematical Statistics with Applications, 7th Ed., by Wackerly, Mendenhall, and Scheaffer.
Problem: 10.81

From two normal populations with respective variances ##\sigma_1^2## and ##\sigma_2^2##, we observe independent sample variances ##S_1^2## and ##S_2^2##, with corresponding degrees of freedom ##\nu_1=n_1-1## and ##\nu_2=n_2-1##. We wish to test ##H_0: \sigma_1^2=\sigma_2^2## versus ##H_a: \sigma_1^2 \neq \sigma_2^2##.

(a) Show that the rejection region given by
$$\{F > F_{\nu_2, \space \alpha/2}^{\nu_1} \space or \space F < (F_{\nu_1, \space \alpha/2}^{\nu_2})^{-1}\}$$
where ##F=S_1^2/S_2^2##, is the same as the rejection region given by
$$\{S_1^2/S_2^2 > F_{\nu_2, \space \alpha/2}^{\nu_1} \space or \space S_2^2/S_1^2 > F_{\nu_1, \space \alpha/2}^{\nu_2}\}.$$

(b) Let ##S_L^2## denote the larger of ##S_1^2## and ##S_2^2## and let ##S_S^2## denote the smaller of ##S_1^2## and ##S_2^2##. Let ##\nu_L## and ##\nu_S## denote the degrees of freedom associated with ##S_L^2## and ##S_S^2##, repectively. Use part (a) to show that, under ##H_0##,
$$P(S_L^2/S_S^2 > F_{\nu_S, \space \alpha/2}^{\nu_L})=\alpha.$$
Note that this gives an equivalent method for testing the equality of two variances.

Homework Equations


N/A

The Attempt at a Solution


[/B]
(a) $$\{F > F_{\nu_2, \space \alpha/2}^{\nu_1} \space or \space F < (F_{\nu_1, \space \alpha/2}^{\nu_2})^{-1}\}$$
$$ = \{S_1^2/S_2^2 > F_{\nu_2, \space \alpha/2}^{\nu_1} \space or \space S_1^2/S_2^2 < (F_{\nu_1, \space \alpha/2}^{\nu_2})^{-1}\}$$
$$ = \{S_1^2/S_2^2 > F_{\nu_2, \space \alpha/2}^{\nu_1} \space or \space (S_1^2/S_2^2)^{-1} > F_{\nu_1, \space \alpha/2}^{\nu_2}\}$$
$$ = \{S_1^2/S_2^2 > F_{\nu_2, \space \alpha/2}^{\nu_1} \space or \space S_2^2/S_1^2 >(F_{\nu_1, \space \alpha/2}^{\nu_2})^{-1}\}$$

(b) I have no idea on, as I'm not entirely certain how the statement could be true in the first place. Because, let's assume that ##S_1^2 = S_L^2## and ##S_2^2 = S_S^2##. Then the problem is saying to show ##P(S_1^2/S_2^2 > F_{\nu_2, \space \alpha/2}^{\nu_1}) = \alpha##, but since this gives the tail probability of the Fisher distribution, and ##F_{\alpha/2}## is defined as the value of F such that the tail probability is ##\frac{\alpha}{2}##, how can ##P(F > F_{\nu_2, \space \alpha/2}^{\nu_1}) = \alpha## when it by definition equals ##\frac{\alpha}{2}##?
 
Physics news on Phys.org
Wait I THINK I may have figured it out. So we have, assuming ##S_1^2 = S_L^2## and ##S_2^2 = S_S^2##:
$$P(F \in Rejection \space Region) = \alpha$$
$$P(S_L^2/S_S^2 > F_{\nu_S, \space \alpha/2}^{\nu_L} \space or \space S_S^2/S_L^2 > F_{\nu_L, \space \alpha/2}^{\nu_S}) = \alpha$$
$$P(S_L^2/S_S^2 > F_{\nu_S, \space \alpha/2}^{\nu_L})+P(S_S^2/S_L^2 > F_{\nu_L, \space \alpha/2}^{\nu_S}) = \alpha$$ since they are mutually exclusive
$$P(S_L^2/S_S^2 > F_{\nu_S, \space \alpha/2}^{\nu_L})+0 = \alpha$$ since ##\frac{S_S^2}{S_L^2} < 1## and F-values are greater than 1 (at least as far as I can tell looking at this table anyway)
$$P(S_L^2/S_S^2 > F_{\nu_S, \space \alpha/2}^{\nu_L})= \alpha$$

Is this right? And if so, could someone give a more intuitive reasoning to this? Because it still feels weird that the tail area is ##\alpha/2## but the probability of being the rejection region is ##\alpha## when it's not possible to be in one tail.
 
Question: A clock's minute hand has length 4 and its hour hand has length 3. What is the distance between the tips at the moment when it is increasing most rapidly?(Putnam Exam Question) Answer: Making assumption that both the hands moves at constant angular velocities, the answer is ## \sqrt{7} .## But don't you think this assumption is somewhat doubtful and wrong?

Similar threads

  • · Replies 2 ·
Replies
2
Views
1K
Replies
1
Views
4K
  • · Replies 1 ·
Replies
1
Views
1K
  • · Replies 20 ·
Replies
20
Views
3K
Replies
8
Views
4K
  • · Replies 6 ·
Replies
6
Views
2K
Replies
8
Views
3K
Replies
3
Views
2K
  • · Replies 46 ·
2
Replies
46
Views
8K
  • · Replies 28 ·
Replies
28
Views
7K